Pelvis

Lakukan tugas rumah & ujian kamu dengan baik sekarang menggunakan Quizwiz!

A potential complication of multiple term gestational births and vaginal deliveries is a prolapsed uterus. To prevent this condition, Kegel exercises may be advised for supporting the uterus.Which pelvic floor muscle is most likely targeted in Kegel exercises? (A) External anal sphincter (B) Bulbospongiosus muscle (C) Obturator internus muscle (D) Pelvic diaphragm (E) Superficial transverse perineal muscle

(D) The pelvic diaphragm, consisting of the levator ani and coccygeus muscles, forms a hammock-like support to the pelvic floor. In females, it supports the bladder, uterus, and rectum. As such, Kegel exercises, which contract and relax pelvic floor muscles, give strength to the pelvic diaphragm in hopes of preventing tears during childbirth.

All of the following bones contribute to the formation of the pelvic cavity EXCEPT (A) ischium (B) pelvis (C) pubis (D) coccyx (E) sacrum

(D) The pelvic girdle is formed by the hip (ischium, pubis, and ilium) bones and the sacrum.

The inferior hypogastric plexus receives fibers from which of the following nerves? (A) pudendal and obturator (B) sciatic and superior gluteal (C) inferior gluteal and lumbosacral (D) pelvic splanchnic and hypogastric (E) sacral sympathetic and obturator

(D) The pelvic splanchnic nerves merge with the hypogastric nerves to form the inferior hypogastric (and pelvic) plexuses.

Which of the following muscles leaves the lesser pelvis through the greater sciatic foramen? (A) pubococcygeus (B) iliococcygeus (C) puborectalis (D) piriformis (E) coccygeus

(D) The piriformis muscle leaves the lesser pelvis through the greater sciatic foramen to attach to the upper border of the greater trochanter of the femur.

After repair of a ruptured diverticulum, a 31-year-old patient begins to spike with fever and complains of abdominal pain. An infection in the deep perineal space would most likely damage which of the following structures? (A) Ischiocavernosus muscles (B) Superficial transverse perineal muscles (C) Levator ani (D) Sphincter urethrae (E) Bulbospongiosus

(D) The sphincter urethrae is found in the deep perineal space, whereas the other structures are located in the superficial perineal space.

The superior end of the vagina surrounds which of the following structures? (A) round ligament (B) urogenital hiatus (C) urethra (D) cervix (E) neck of bladder

(D) The superior end of the vagina surrounds the cervix; the lower end passes anteroinferiorly through the pelvic floor to open in the vestibule.

All of the following muscles compress the vagina and act like sphincters EXCEPT (A) pubovaginalis (B) external urethral sphincter (C) urethrovaginal sphincter (D) bulbospongiosus (E) pubococcygeus

(E) The pubovaginalis, external urethral sphincter, urethrovaginal sphincter, and bulbospongiosus compress the vagina and act like sphincters.

The pudendal nerve innervates which of the following structures? (A) ovaries (B) testes (C) uterus (D) levator ani (E) muscles of the deep and superficial perineal pouch

(E) The pudendal nerve supplies most of the innervation to the perineum. Toward the distal end of the pudendal canal, the pudendal nerve splits, giving rise to the perineal nerves and continuing as the dorsal nerve of the penis or clitoris.

Which of the following arteries arises from the anterior division of the internal iliac? (A) superior rectal (B) iliolumbar (C) superior gluteal (D) gonadal (E) obturator

(E) The superior gluteal and iliolumbar arteries arise from the posterior division of the internal iliac arteries. The gonadal arteries arise from the abdominal aorta and the superior rectal arteries from the inferior mesenteric artery. The umbilical, obturator, uterine, vaginal, internal pudendal, and inferior gluteal arteries arise from the anterior division of the internal iliac arteries.

Which of the following nerves exits the pelvis through the greater sciatic foramen, superior to the piriformis? (A) sciatic (B) pudendal (C) superior gluteal (D) lumbosacral trunk (E) obturator

(C) The superior gluteal nerve arises from the posterior divisions of the ventral rami of L4 through S1 and leaves the pelvis through the greater sciatic foramen, superior to the piriformis.

The anal columns contain which of the following structures? (A) internal pudendal vessels (B) pudendal nerve (C) superior rectal vessels (D) obturator nerve (E) inferior rectal vessels

(C) The superior half of the anal canal contains mucous membrane that is characterized by a series of longitudinal ridges called anal columns. These columns contain the terminal branches of the superior rectal artery and vein.

Distally the corpus spongiosum expands to form which of the following structures? (A) bulb of the penis (B) clitoris (C) vestibule (D) glans penis (E) crura of the penis

(D) Distally the corpus spongiosum expands to form the conical glans penis.

The paramesonephric ducts in the male degenerate except for which of the following structures? (A) efferent ductules of the testis (B) appendix testis (C) ductus epididymis (D) seminal vesicles (E) ductus deferens

(B) Except for the most cranial portion, the appendix epididymis, the mesonephric ducts persist and form the main genital ducts. Immediately below the entrance of the efferent ductules, the mesonephric ducts elongate and become highly convoluted, forming the ductus epididymis. From the tail of the epididymis to the outbudding of the seminal vesicle, the mesonephric ducts obtain a thick muscular coat and form the ductus deferens. The region of the ducts beyond the seminal vesicles is the ejaculatory duct. Except for a small portion at their cranial ends, the appendix testis, the paramesonephric ducts in the male degenerate.

In males the rectum is related anteriorly to all of the following structures EXCEPT (A) fundus of the urinary bladder (B) proximal parts of the ureters (C) ductus deferens (D) seminal vesicles (E) prostate

(B) In males the rectum is related anteriorly to the fundus of the urinary bladder, terminal parts of the ureters, ductus deferens, seminal vesicles, and prostate.

Which of the following structures may be referred to as the fourchette? (A) prepuce of the clitoris (B) frenulum of the labia minora (C) vestibule of the vagina (D) glans clitoris (E) mons pubis

(B) In young women, especially virgins, the labia minora are connected by a small fold known as the frenulum of the labia minora or the fourchette.

Parasympathetic stimulation in the female produces which of the following? (A) decreased vaginal secretions (B) engorgement of erectile tissue in the bulbs of the vestibule (C) engorgement of the greater vestibular gland (D) decrease in size of the clitoris (E) the clitoris becomes flaccid

(B) Parasympathetic stimulation in the female produces an increase in vaginal secretions, erection of the clitoris, and engorgement of erectile tissue in the bulbs of the vestibule.

A 25-year-old medical student in good health develops severe pain in the area around her umbilicus. She complains of nausea and is taken to the emergency department. While there, the pain becomes more localized in the lower right quadrant of her abdomen and the physician diagnoses appendicitis. Which of the following nerves perceived pain in the area around the umbilicus and most likely carried the pain sensations to the central nervous system? (A) Inferior hypogastric nerves (B) Lesser splanchnic nerves (C) Pudendal nerves (D) Superior hypogastric nerves (E) Vagus nerves

(B) The T10 dermatome is associated with the umbilical region. Sensory neurons course from the umbilical skin to the T10 spinal cord level. Visceral sensory neurons course from the appendix to the T10 spinal cord level as well as via the lesser splanchnic nerves. Therefore, the referred pain comes from the lesser splanchnic nerves.

The oocytes expelled from the ovaries usually are fertilized in which of the following areas of the uterine tubes? (A) infundibulum (B) ampulla (C) isthmus (D) uterine (E) fimbria

(B) The ampulla is the widest and longest part of the uterine tubes. It begins at the medial end of the infundibulum. Oocytes expelled from the ovaries are usually fertilized in the ampulla.

Laterally, the peritoneum of the broad ligament is prolonged superiorly over the ovarian vessels as which of the following structures? (A) mesovarium (B) mesosalpinx (C) mesometrium (D) suspensory ligament of the ovary (E) transverse cervical (cardinal) ligaments

(D) Laterally, the peritoneum of the broad ligament is prolonged superiorly over the vessels as the suspensory ligament of the ovary. The part of the broad ligament by which the ovary is suspended is the mesovarium. The part of the broad ligament forming the mesentery of the uterine tube is the mesosalpinx. The major part of the broad ligament, the mesentery of the uterus, or mesometrium, is below the mesosalpinx and mesovarium.

Lithotripsy uses shock waves to break up which of the following structures? (A) blood clots (B) foreign bodies (C) small tumors (D) urinary calculi (E) abscesses

(D) Lithotripsy uses shock waves to break up a stone into small fragments that can be passed in the urine.

A radiologist interprets a lymphangiogram for a 29-year-old patient with metastatic carcinoma. Upper lumbar nodes most likely receive lymph from which of the following structures? (A) Lower part of the anal canal (B) Labium majus (C) Clitoris (D) Testis (E) Scrotum

(D) Lymphatic vessels from the testis and epididymis ascend along the testicular vessels in the spermatic cord through the inguinal canal and continue upward in the abdomen to drain into the upper lumbar nodes. The lymph from the other structures drains into the superficial inguinal lymph nodes.

Each of the following statements relating to the labia minora is correct EXCEPT (A) They are folds of fat. (B) They are hairless. (C) They are enclosed in the pudendal cleft within the labia majora. (D) They have a core of spongy connective tissue containing erectile tissue. (E) They contain many sebaceous glands and sensory nerve endings.

(A) The labia minora are folds of fat-free, hairless skin. They are enclosed in the pudendal cleft within the labia majora, immediately surrounding the vestibule of the vagina. They have a core of spongy connective tissue containing erectile tissue and many small blood vessels.

The navicular fossa is located in which of the following structures? (A) glans penis (B) prostate (C) bladder (D) seminal vesicle (E) expanded in the clitoris

(A) The lumen of the spongy urethra is approximately 5 mm in diameter and expanded in the bulb of the penis to form the intrabulbar fossa and in the glans penis to form the fossa navicularis.

The mesosalpinx forms the mesentery for which of the following structures? (A) uterine tube (B) ovary (C) small intestine (D) bladder (E) uterus

(A) The part of the broad ligament forming the mesentery of the uterine tube is the mesosalpinx.

All of the following structures provide boundaries for the pelvic inlet EXCEPT (A) inferior ramus of the pubis (B) sacral promontory (C) anterior border of the ala of the sacrum (D) arcuate line of the ilium (E) pecten pubis

(A) The pelvic inlet is bounded by the superior margin of the pubic symphysis anteriorly, posterior border of the pubic crest, pecten pubis, arcuate line of the ilium, anterior border of the ala of the sacrum, and the sacral promontory.

Which of the following arteries is considered to be the artery of the pelvis? (A) obturator (B) pudendal (C) uterine (D) internal iliac (E) sacral

(D) The internal iliac artery is the artery of the pelvis; however, it does supply branches to the buttocks, thighs, and the perineum

A 29-year-old carpenter sustains severe injuries of the pelvic splanchnic nerve by a deep puncture wound, which has become contaminated. The injured parasympathetic preganglionic fibers in the splanchnic nerve are most likely to synapse in which of the following ganglia? (A) Ganglia in or near the viscera or pelvic plexus (B) Sympathetic chain ganglia (C) Collateral ganglia (D) Dorsal root ganglia (E) Ganglion impar

(A) The pelvic splanchnic nerves carry preganglionic parasympathetic general visceral efferent fibers that synapse in the ganglia of the inferior hypogastric plexus and in terminal ganglia in the muscular walls of the pelvic organs. The sympathetic preganglionic fibers synapse in the sympathetic chain (paravertebral) ganglia or in the collateral (prevertebral) ganglia. The dorsal root ganglia contain cell bodies of general somatic afferent (GSA) and general visceral afferent (GVA) fibers and have no synapsis. The two sympathetic trunks unite and terminate in the ganglion impar (coccygeal ganglion), which is the most inferior, unpaired ganglion located in front of the coccyx.

Which of the following structures is incised during median episiotomy for childbirth? (A) perineal body (B) greater vestibular glands (C) lesser vestibular glands (D) clitoris (E) urethra

(A) The perineal body is the major structure incised during median episiotomy for childbirth.

The rectosigmoid junction lies anterior to which of the following structures? (A) S3 vertebra (B) prostate (C) obturator foramen (D) bladder (E) pararectal fossae

(A) The rectosigmoid junction lies anterior to the S3 vertebra.

Which of the following structures is located between the ischial spine and the ischial tuberosity? (A) obturator foramen (B) lesser sciatic notch (C) acetabular notch (D) pubic arch (E) arcuate line

(B) The concavity between the ischial spine and the ischial tuberosity is the lesser sciatic notch. The larger concavity, the greater sciatic notch, is superior to the ischial spine and is formed in part by the ilium.

When a person is in the anatomical position, which of the following structures lie in the same vertical plane? (A) sacral promontory and pubic tubercles (B) anterior superior iliac spines and the anterior aspect of the pubic symphysis (C) posterior superior iliac spines and the posterior aspect of the ischial tuberosity (D) ischial spines and the posterior border of the obturator foramen (E) superior pubic rami and the greater sciatic notch

(B) When a person is in the anatomical position, the anterior superior iliac spines and the anterior aspect of the pubic symphysis lie in the same vertical plane.

A 31-year-old woman delivers her first child via natural childbirth, without anesthetics. During the delivery, she experiences painful spasms in the muscles in the medial part of her right thigh and paresthesia in the skin in the medial thigh. Following delivery, these conditions gradually dissipate. Which of the following nerves may have been compressed during this childbirth? (A) Inferior gluteal nerve (B) Pudendal nerve (C) Obturator nerve (D) Sciatic nerve (E) Pelvic splanchnic nerves

(C)

Persons with spondylolysis have a defect in which of the following structures? (A) vertebral arch (B) body of L5 vertebra (C) sacrum (D) vertebral canal (E) vertebral body

(A) Persons with spondylolysis have a defect in the vertebral arch. When this is bilateral, it results in the L5 vertebra being divided into two pieces. If the parts separate, the abnormality is spondylolisthesis, which is anterior displacement of the body of the L5 vertebra on the sacrum.

The perineal body is the site of convergence of all of the following muscles EXCEPT (A) ischiocavernous (B) bulbospongiosus (C) superficial transverse perineal (D) deep transverse perineal (E) external anal sphincter

(A) The bulbospongiosus, external anal sphincter, and superficial and deep transverse perineal muscles converge at the site of the perineal body.

A pregnant woman in active labor receives an epidural anesthetic to relieve pain from her uterine contractions and cervical dilation in preparation for the birth of her child. Pain derived from the labor contractions of the upper uterus is referred to what spinal cord level? (A) T6 (B) T12 (C) L4 (D) S1 (E) S3

(B)

A 37-year-old woman complains of a bearing-down sensation in her womb and an increased frequency of and burning sensation on urination. On examination by her gynecologist, she is diagnosed with a uterine prolapse. Which of the following structures provides the primary support for the cervix of the uterus? (A) External anal sphincter (B) Broad ligament of the uterus (C) Cardinal (transverse cervical) ligament (D) Round ligament of the uterus (E) Suspensory ligament of the ovary

(C) The cardinal (transverse cervical) ligament provides the major ligamentous support for the uterus. The sphincter ani externus does not support the uterus. The broad and round ligaments of the uterus provide minor supports for the uterus. The suspensory ligament of the ovary does not support the uterus.

The inferior ends of the seminal vesicles are closely related to the rectum and are separated from it only by which of the following structures? (A) pubovesical ligament (B) puboprostatic ligament (C) rectovesical septum (D) puborectalis muscle (E) coccygeus muscle

(C) The inferior ends of the seminal vesicles are closely related to the rectum and are separated from it only by the rectovesical septum, a membranous partition.

The median umbilical ligament contains which of the following structures? (A) inferior epigastric vessels (B) obturator vessels (C) remnant of the urachus (D) uterine tubes (E) ovarian vessels

(C) The median umbilical ligament is formed by the remnant of the urachus.

The only parts remaining from the mesonephric system in the female include which of the following structures? (A) fimbriae (B) uterine tubes (C) epoöphoron (D) uterus (E) cervix

(C) The only parts remaining from the mesonephric system are the epoöphoron, paroöphoron, and Gartner's cyst.

All of the following osseofibrous structures mark the boundaries of the perineum EXCEPT (A) pubic symphysis (B) inferior pubic rami (C) sacrospinous ligament (D) ischial tuberosities (E) ischial rami

(C) The osseofibrous structures marking the boundaries of the perineum include the pubic symphysis, inferior pubic rami, ischial rami, ischial tuberosities, sacrotuberous ligaments, inferiormost sacrum, and coccyx.

While performing a newborn checkup on a baby boy, a pediatrician discovers the external urethral orifice (urethral opening) is located on the ventral (bottom) surface of the penis. What is the most likely diagnosis in this newborn? (A) Epispadias (B) Hypospadias (C) Micropenis (D) Exstrophy of the bladder (E) Cryptorchidism

(B)

A physician examining a 16-year-old girl for the fi rst time notes that she is in good overall health and appears outwardly to have reached puberty. However, she explains that she has not yet had a menstrual period. Full physical and radiological examinations reveal a short, closed vaginal pouch, and no upper vagina, cervix, or uterus. Which of the following descriptions is the most likely cause of this condition? (A) Atresia of the paramesonephric ducts (B) Atresia of the mesonephric ducts (C) Atresia of the urogenital sinus (D) Imperforate hymen (E) Agenesis of the ureteric bud

(A)

An abscess is detected between the perineal membrane and the inferior fascia of the pelvic diaphragm in a 27-year-old man. Which of the following structures would most likely be affected by this abscess within the deep perineal space? (A) Bulbourethral glands (B) Perineal body (C) Superfi cial transverse perineal muscles (D) Bulbospongiosus muscles (E) Ischiocavernosus muscles

(A)

55-year-old man who has alcoholic cirrhosis of the liver is brought to the emergency department because he has been vomiting blood for the past 2 hours. He has a 2-month history of abdominal distention, dilated veins over the anterior abdominal wall, and internal hemorrhoids. Which of the following veins is the most likely origin of the hematemesis? (A) Esophageal veins (B) Inferior mesenteric veins (C) Paraumbilical veins (D) Superior mesenteric vein (E) Superior vena cava

(A) A cirrhotic liver prevents all portal blood from flowing through the liver sinusoids. Therefore, portal hypertension occurs with blood backing up to the sites of portocaval anastomoses, including the esophageal veins. Chronic portal hypertension will result in swelling of the esophageal veins and potential hemorrhaging, causing hematemesis.

The perineum is divided into two triangles by drawing a transverse line between which of the following structures? (A) anterior ends of the ischial tuberosities (B) coccyx to pubic tubercles (C) inferior iliac spines to pubic symphysis (D) medial ends of inguinal ligament to tip of coccyx (E) sacrum to pubic symphysis

(A) A transverse line joining the anterior ends of the ischial tuberosities divides the perineum into two triangles. The anal triangle, containing the anus, is posterior to this line. The urogenital triangle, containing the root of the scrotum and penis in males and the external genitalia in females is anterior to this line.

Which of the following muscles passes through the lesser sciatic foramen? (A) obturator internus (B) piriformis (C) puborectalis (D) pubococcygeus (E) iliococcygeus

(A) Each obturator internus passes posteriorly from the lesser pelvis through the lesser sciatic foramen and turns sharply laterally to attach to the greater trochanter of the femur.

A 16-year-old boy presents to the emergency department with rupture of the penile urethra. Extravasated urine from this injury can spread into which of the following structures? (A) Scrotum (B) Ischiorectal fossa (C) Pelvic cavity (D) Testis (E) Thigh

(A) Extravasated urine from the penile urethra below the perineal membrane spreads into the superficial perineal space, scrotum, penis, and anterior abdominal wall. However, it does not spread into the testis, ischiorectal fossa, pelvic cavity, and thigh because Scarpa's fascia ends by firm attachment to the fascia lata of the thigh.

Parasympathetic fibers to the bladder are derived from which of the following nerves? (A) pelvic splanchnic (B) greater splanchnic (C) T11-L2 (D) superior hypogastric plexus (E) sacral plexus

(A) Parasympathetic fibers to the bladder are derived from the pelvic splanchnic nerves. They are motor to the detrusor muscle and inhibitory to the internal sphincter. Sympathetic fibers to the bladder are derived from T11 through L2.

All of the following nerves provide innervation to the scrotum EXCEPT (A) obturator (B) ilioinguinal (C) genitofemoral (D) pudendal (E) posterior femoral cutaneous

(A) The anterior aspect of the scrotum is supplied by anterior scrotal nerves derived from the ilioinguinal and the genital branch of the genitofemoral nerve. The posterior aspect of the scrotum is supplied by posterior scrotal nerves, provided by perineal branches of the pudendal and perineal branches of the posterior femoral cutaneous nerve.

Which of the following muscles covers and pads the lateral pelvic wall? (A) obturator internus (B) piriformis (C) pubococcygeus (D) iliococcygeus (E) ischiococcygeus

(A) The bony framework of the lateral pelvic walls is formed by the hip bones and the obturator foramen, which is closed by the obturator membrane. The obturator internus muscles cover and thus pad most of the lateral pelvic walls.

A 22-year-old man has a gonorrheal infection that has infiltrated the space between the inferior fascia of the urogenital diaphragm and the superficial perineal fascia. Which of the following structures might be inflamed? (A) Bulb of the penis (B) Bulbourethral gland (C) Membranous part of the male urethra (D) Deep transverse perineal muscle (E) Sphincter urethrae

(A) The bulb of the penis is located in the superficial perineal space between the inferior fascia of the urogenital diaphragm and the membranous layer of the superficial perineal fascia (Colles's fascia). All of the other structures are found in the deep perineal pouch.

A 58-year-old man is diagnosed as having a slowly growing tumor in the deep perineal space. Which of the following structures would most likely be injured? (A) Bulbourethral glands (B) Crus of penis (C) Bulb of vestibule (D) Spongy urethra (E) Great vestibular gland

(A) The deep perineal space contains the bulbourethral (Cowper's) glands. The crus of the penis, bulb of the vestibule, spongy urethra, and great vestibular gland are found in the superficial perineal space.

A first-year resident in the urology department reviews pelvic anatomy before seeing patients. Which of the following statements is correct? (A) The dorsal artery of the penis supplies the glans penis. (B) The seminal vesicles store spermatozoa. (C) The duct of the bulbourethral gland opens into the membranous urethra. (D) The duct of the greater vestibular gland opens into the vagina. (E) The anterior lobe of the prostate gland is prone to carcinomatous transformation.

(A) The dorsal artery of the penis supplies the glans penis. The seminal vesicles store no spermatozoa. The duct of the bulbourethral gland opens into the bulbous portion of the spongy urethra, whereas the greater vestibular gland opens into the vestibule between the labium minora and the hymen. The anterior lobe of the prostate is devoid of glandular substance, the middle lobe is prone to benign hypertrophy, and the posterior lobe is prone to carcinomatous transformation.

All of the following statements concerning the ductus deferens are correct EXCEPT (A) It begins in the head of the epididymis. (B) It ascends in the spermatic cord. (C) It passes through the inguinal canal. (D) It joins the duct of the seminal vesicle to form the ejaculatory duct. (E) It descends medial to the ureter and seminal vesicle.

(A) The ductus deferens begins in the tail of the epididymis and ascends in the spermatic cord. It passes through the inguinal canal and enters the pelvis. It ends by joining the duct of the seminal vesicle to form the ejaculatory duct. It descends medial to the ureter and seminal vesicle.

A 52-year-old man undergoes surgery to biopsy iliac lymph nodes. The physician tells the patient that it is important to identify the peripheral spinal nerves to protect them from being damaged during the surgery. The most likely location to find the genitofemoral nerve is coursing along which of the following surfaces? (A) Anterior surface of the psoas major muscle (B) Anterior surface of the quadratus lumborum muscle (C) Inferior surface of the iliacus muscle (D) Inferior surface of rib 12 (E) Medial surface of the quadratus lumborum muscle (F) Medial surface of psoas major muscle

(A) The genitofemoral nerve courses along the anterior surface of the psoas major muscle.

Collateral circulation between the subclavian and external iliac arteries is created by an anastomosis between which of the following structures? (A) Epigastric arteries (B) Lumbar arteries (C) Posterior intercostal arteries (D) Round ligament of the liver (E) Superficial epigastric arteries

(A) The inferior epigastric artery branches off the external iliac artery and forms an anastomosis with the superior epigastric artery on the posterior surface of the rectus abdominis muscle. The superior epigastric artery branches off the internal thoracic artery, a branch of the subclavian artery.

The ischiopubic or conjoint ramus is formed when the ischial ramus joins which of the following structures? (A) Inferior pubic ramus (B) Ischial spine (C) Pubic symphysis (D) Pubic tubercle (E) Superior pubic ramus (F) Sciatic notch

(A) The inferior pubic ramus joins the ischial ramus along the inferior aspect of the os coxa to form the ischiopubic ramus.

A 21-year-old marine biologist asks about her first bimanual examination, and it is explained to her that the normal position of the uterus is: (A) Anteflexed and anteverted (B) Retroflexed and anteverted (C) Anteflexed and retroverted (D) Retroverted and retroflexed (E) Anteverted and retroverted

(A) The normal position of the uterus is anteverted (i.e., angle of 90 degrees at the junction of the vagina and cervical canal) and anteflexed (i.e., angle of 160 to 170 degrees at the junction of the cervix and body).

The paraurethral glands open into which of the following structures? (A) near the external urethral orifice (B) in the neck of the bladder (C) in the prostatic utricle (D) in the seminal colliculus (E) in the ejaculatory ducts

(A) The paraurethral glands are homologues to the prostate. They have common paraurethral ducts, which open, one on each side, near the external urethral orifice.

A 43-year-old woman presents with a prolapsed uterus. Repair of a prolapsed uterus requires knowledge of the supporting structures of the uterus. Which of the following structures plays the most important role in the support of the uterus? (A) Levator ani (B) Sphincter urethrae (C) Uterosacral ligament (D) Ovarian ligament (E) Arcuate pubic ligament

(A) The pelvic diaphragm, particularly the levator ani, provides the most important support for the uterus, although the urogenital diaphragm and the uterosacral and ovarian ligaments support the uterus. The arcuate pubic ligament arches across the inferior aspect of the pubic symphysis.

While performing a pelvic exenteration, the surgical oncologist notices a fractured or ruptured boundary of the pelvic inlet. Which of the following structures is most likely damaged? (A) Promontory of the sacrum (B) Anterior-inferior iliac spine (C) Inguinal ligament (D) Iliac crest (E) Arcuate pubic ligament

(A) The pelvic inlet (pelvic brim) is bounded by the promontory and the anterior border of the ala of the sacrum, the arcuate line of the ilium, the pectineal line, the pubic crest, and the superior margin of the pubic symphysis.

A 72-year-old man comes to his physician for an annual checkup. Which of the following structures is most readily palpated during rectal examination? (A) Prostate gland (B) Epididymis (C) Ejaculatory duct (D) Ureter (E) Testis

(A) The prostate gland may be palpated on rectal examination. The ejaculatory duct runs within the prostate gland and cannot be felt. In the male, the pelvic part of the ureter lies lateral to the ductus deferens and enters the posterosuperior angle of the bladder, where it is situated anterior to the upper end of the seminal vesicle, and thus, it cannot be palpated during rectal examination. However, in the female, the ureter can be palpated during vaginal examination because it runs near the uterine cervix and the lateral fornix of the vagina to enter the posterosuperior angle of the bladder. The testes are examined during a routine annual checkup but obviously not during a rectal examination.

Which of the following statements concerning the sacral sympathetic trunks is correct? (A) Usually has four sympathetic ganglia. (B) Descends on the pelvic surface of the ischium. (C) Ends as the dorsal nerve of the penis or clitoris. (D) Passes through the obturator canal. (E) Pierces the sacrotuberous ligament.

(A) The sacral sympathetic trunks descend posterior to the rectum in the extraperitoneal connective tissue and send gray rami communicantes to each of the ventral rami of the sacral and coccygeal nerves. Each of the sacral trunks is smaller than the lumbar trunks and usually has four sympathetic ganglia.

The lateral part of the superior ramus of the pubis forms which of the following structures? (A) iliopubic eminence (B) pubic tubercle (C) pecten pubis (D) anterior inferior iliac spine (E) acetabulum

(C) The pubis is an angulated bone with a superior ramus that helps to form the acetabulum and an inferior ramus that helps to form the obturator foramen. A thickening on the anterior part of the body of the pubis is the pubic crest, which ends laterally as a prominent bump, the pubic tubercle. The lateral part of the superior ramus has an oblique ridge known as the pecten pubis or pectineal line of the pubis.

The pudendal canal is a space within which of the following structures? (A) the deep perineal space (B) the superficial perineal space (C) the obturator fascia (D) the broad ligament (E) the urogenital triangle

(C) The pudendal canal is a space within the obturator fascia, which covers the medial aspect of the obturator internus and lines the lateral wall of the ischioanal fossa.

A 30-year-old woman sustains a stage 4 tear in the perineum during a difficult delivery. In preparation to repair the tear, an anesthetic nerve block is administered to the pudendal nerve as it courses around the sacrospinous ligament. Which of the following areas is most likely blocked by the anesthetic? (A) L2-L4 cutaneous field (B) S1 cutaneous field (C) S2-S4 cutaneous field (D) L2-L4 dermatomes (E) S1 dermatome (F) S2-S4 dermatomes

(C) The pudendal nerve carries sensory axons from the genital region to the S2-S4 spinal cord levels. Therefore, it supplies sensory distribution for a region of the S2, S3, and S4 dermatomes, but not all of the parts of each dermatome. Therefore, the anesthetic blocked a cutaneous field, not a dermatome.

Three days after giving birth, a 32-year-old woman develops a fever and right lower abdominal pain. Ultrasonography shows a right ovarian vein thrombosis extending proximally. The thrombus most likely extends into the (A) ascending lumbar vein (B) hepatic portal vein (C) inferior vena cava (D) renal vein (E) right internal iliac vein

(C) The right ovarian vein courses from the right ovary to the inferior vena cava. Therefore, if the thrombosis extends proximally, it will course into the inferior vena cava. If the thrombosis were in the left ovarian vein, it would extend into the left renal vein.

A 37-year-old small business manager receives a gunshot wound in the pelvic cavity, resulting in a lesion of the sacral splanchnic nerves. Which of the following nerve fibers would primarily be damaged? (A) Postganglionic parasympathetic fibers (B) Postganglionic sympathetic fibers (C) Preganglionic sympathetic fibers (D) Preganglionic parasympathetic fibers (E) Postganglionic sympathetic and parasympathetic fibers

(C) The sacral splanchnic nerves consist primarily of preganglionic sympathetic neurons and also contain GVA fibers. None of the other fibers listed are contained in these nerves.

All of the following statements concerning the pudendal nerve are correct EXCEPT (A) It is derived from the anterior divisions of the ventral rami of S2 through S4. (B) It accompanies the internal pudendal artery. (C) It leaves the pelvis through the greater sciatic foramen. (D) It leaves the pelvis through the greater sciatic foramen inferior to the piriformis and coccygeus muscles. (E) It is the chief sensory nerve of the external genitalia.

(D) The pudendal nerve is derived from the anterior divisions of the ventral rami of S2 through S4. It accompanies the internal pudendal artery and leaves the pelvis through the greater sciatic foramen between the piriformis and coccygeus muscles. It is the main nerve of the perineum and the chief sensory nerve of the external genitalia.

A 37-year-old man is suffering from carcinoma of the skin of the penis. Cancer cells are likely to metastasize directly to which of the following lymph nodes? (A) External iliac nodes (B) Internal iliac nodes (C) Superficial inguinal nodes (D) Aortic (lumbar) nodes (E) Deep inguinal nodes

(C) The superficial inguinal nodes receive lymph from the penis, scrotum, buttocks, labium majus, and the lower parts of the vagina and anal canal. These nodes have efferent vessels that drain primarily into the external iliac and common iliac nodes and ultimately to the lumbar (aortic) nodes. The internal iliac nodes receive lymph from the upper part of the rectum, vagina, uterus, and other pelvic organs, and they drain into the common iliac nodes and then into the lumbar (aortic) nodes. Lymph vessels from the glans penis drain initially into the deep inguinal nodes and then into the external iliac nodes.

On a busy Saturday night in Chicago, a 16-year-old boy presents to the emergency department with a stab wound from a knife that entered the pelvis above the piriformis muscle. Which of the following structures is most likely to be damaged? (A) Sciatic nerve (B) Internal pudendal artery (C) Superior gluteal nerve (D) Inferior gluteal artery (E) Posterior femoral cutaneous nerve

(C) The superior gluteal nerve leaves the pelvis through the greater sciatic foramen, above the piriformis. The sciatic nerve, internal pudendal vessels, inferior gluteal vessels and nerve, and posterior femoral cutaneous nerve leave the pelvis below the piriformis.

Which of the following arteries gives rise to the deferential artery? (A) internal pudendal (B) testicular (C) inferior vesicle (D) obturator (E) umbilical

(C) The tiny deferential artery usually arises from the inferior vesical artery and terminates by anastomosing with the testicular artery, posterior to the testis.

A 5-month-old boy is admitted to the children's hospital because of urine being expelled from the dorsal aspect of the penis. Which of the following embryologic structures failed to fuse in this patient? (A) Labioscrotal swellings or folds (B) Urogenital sinus (C) Spongy urethra (D) Phallus (E) Urethral folds

(C) A developmental defect in the spongy urethra results in epispadias, causing the patient to pass urine through an opening on the dorsum of the penis. Labioscrotal swellings form the scrotum in males and the labia majora in females. Urogenital sinus forms the urinary bladder, urethra, prostate, and bulbourethral glands in males, and the bladder, urethra, lower vagina, and greater vestibular glands in females. The phallus (genital tubercle) forms the penis in males and the clitoris in females. Urethral (urogenital) folds form the spongy urethra and a portion of the shaft of the penis in males and the labia minora in females.

A 42-year-old man has a vasectomy. The physician explains to him that 3 to 4 months after the procedure, when he has an orgasm during sexual intercourse, most likely he will (A) no longer produce an ejaculate (B) still produce an ejaculate and the ejaculate will contain sperm (C) still produce an ejaculate but the ejaculate will not contain sperm

(C) A vasectomy (a surgical procedure in which the ductus deferens is cut for the purpose of sterilization) will eventually sterilize the male by inhibiting sperm from entering the ejaculate. However, seminal contributions from the seminal vesicles, prostate, and bulbourethral gland will continue. Therefore, ejaculation will result in an ejaculate but without any sperm.

Which of the following terms correctly applies to the pelvis of the normal female? (A) spondyloid (B) android (C) gynecoid (D) anthropoid (E) platypelloid

(C) Android and anthropoid pelves are common in males. The platypelloid pelvis is uncommon in both males and females. The gynecoid pelvis is the normal female type of pelvis.

A 78-year-old man has carcinoma of the rectum. The cancer is likely to metastasize via the veins into which of the following structures? (A) Spleen (B) Kidney (C) Liver (D) Duodenum (E) Suprarenal gland

(C) Cancer cells from rectal cancer are likely to metastasize to the liver via the superior rectal, inferior mesenteric, splenic, and portal veins. Cancer cells are not directly spread to the other organs listed. The spleen and duodenum drain their venous blood to the portal venous system, and the kidney and suprarenal gland empty into the caval (inferior vena cava) system.

During sexual arousal, an erection is caused by a dilation of arteries filling the erectile tissue of the penis. Innervation of the penile arteries is provided by which of the following nerves? (A) Genitofemoral nerves (B) Ilioinguinal nerves (C) Pelvic splanchnic nerves (D) Pudendal nerves (E) Sacral splanchnic nerves

(C) Dilation of penile arteries resulting in blood filling erectile tissue is under parasympathetic innervation. Therefore, the pelvic splanchnic nerves are responsible for transporting parasympathetic nerves to the penile arteries. The genitofemoral, ilioinguinal, and pudendal nerves are all somatic and do not cause an erection. The sacral splanchnics are responsible for transporting the sympathetics and will result in ejaculation. Remember, "point" and "shoot" ("p" parasympathetic; "s" sympathetic).

The relationship, "water passing under the bridge," is an especially important one for surgeons ligating which of the following arteries? (A) ovarian (B) testicular (C) uterine (D) vaginal (E) obturator

(C) In its uppermost portion, at the base of the peritoneal broad ligament, the uterine artery runs transversely toward the cervix while the ureters pass immediately beneath them as they pass on each side of the cervix toward the bladder. This relationship, "water passing under the bridge," is an especially important one for surgeons ligating the uterine artery, as in a hysterectomy.

During pelvic surgery, a surgeon notices severe bleeding from the artery that remains within the true pelvis. Which of the following arteries is most likely to be injured? (A) Iliolumbar artery (B) Obturator artery (C) Uterine artery (D) Internal pudendal artery (E) Inferior gluteal artery

(C) Of all the arteries listed, the uterine artery remains within the pelvic cavity.

Which of the following nerves innervates the lower one fourth of the vagina? (A) pelvic splanchnics (B) lumbar splanchnics (C) pudendal (D) superior hypogastric plexus (E) inferior hypogastric plexus

(C) Only the lower one-fifth to one-fourth of the vagina is somatic in terms of innervation. The innervation of this part of the vagina is from the deep perineal branch of the pudendal nerve.

Sweat glands within the S2 dermatome along the posterior region of the thigh most likely receive innervation via preganglionic sympathetic neurons originating from which of the following central nervous system levels? (A) Brainstem (B) C2 spinal cord level (C) L2 spinal cord level (D) S2 spinal cord level (E) T2 spinal cord level

(C) Preganglionic sympathetic neurons originate between the T1 and L2 spinal cord levels. Dermatomes within the sacral region, such as the S2 dermatome described in this question, are supplied by sympathetics from the L2 spinal cord level, the lowest of all sympathetic innervation origin.

The external oblique, internal oblique, and transversus abdominis aponeuroses all have a common insertion into which structure? (A) Arcuate line (B) Inguinal ligament (C) Linea alba (D) Pectineal line (E) Pubic tubercle

(C) The aponeuroses from the external oblique, internal oblique, and transversus abdominis muscles create the rectus sheath and then insert on the linea alba between the two rectus abdominis muscles.

Helicine arteries are located in which of the following structures? (A) superficial perineal space (B) deep perineal space (C) corpora cavernosa (D) spermatic cord (E) bulbospongiosum

(C) The deep arteries of the penis are the main vessels supplying the cavernous spaces in the erectile tissue of the corpora cavernosa and are therefore involved in the erection of the penis. They give off numerous branches that open directly into the cavernous spaces. When the penis is flaccid, these arteries are coiled and therefore are called helicine arteries.

The distal end of the ovary connects to the lateral wall of the pelvis by which of the following structures? (A) round ligament (B) ligament of the ovary (C) suspensory ligament of the ovary (D) transverse cervical ligament (E) lateral cervical ligament

(C) The distal end of the ovary connects to the lateral wall of the pelvis by the suspensory ligament of the ovary. The ligament conveys the ovarian vessels, lymphatics, and nerves to and from the ovary and constitutes the lateral part of the mesovarium of the broad ligament. The ovary also attaches to the uterus by the ligament of the ovary.

Which of the following structures is located at the free anterior borders of the levator ani? (A) coccygeus muscle (B) piriformis muscle (C) urogenital hiatus (D) obturator internus muscle (E) rectum

(C) The free anterior borders of the levator ani are separated by a gap, the urogenital hiatus, through which the urethra (and, in the female, the vagina) passes.

The membranous part of the male urethra is located in which of the following structures? (A) bladder (B) prostate (C) external urethral sphincter (D) bulb of penis (E) glans penis

(C) The intermediate part of the urethra (membranous part) is the section passing through the external urethral sphincter and the perineal membrane. The short intermediate part, extending from the prostatic urethra to the spongy urethra, is the narrowest and least distensible part of the urethra.

A 17-year-old girl is brought to a refugee camp and has significant blood loss. She recently underwent a form of genital mutilation called excision, where the clitoris and labia minora were removed. Direct branches of which of the following arteries are most likely responsible for the blood loss? (A) External iliac (B) Inferior rectal (C) Internal pudendal (D) Ovarian (E) Uterine

(C) The internal pudendal artery supplies all of the perineum, including the clitoris and labia minora.

Which of the following structures opens into the prostatic sinus? (A) prostatic utricle (B) ejaculatory ducts (C) prostatic ductules (D) seminal vesicles (E) bulbourethral glands

(C) The internal surface of the posterior wall of the prostatic urethra has a median ridge known as the urethral crest. A groove on each side, known as the prostatic sinus, receives the prostatic ductules. A rounded eminence located in the middle of the median ridge is known as the seminal colliculus. The prostatic utricle is an embryonic remnant of the uterus and part of the vagina. The ejaculatory ducts open beside the prostatic utricle.

A neurosurgeon performs surgical resection of a rare meningeal tumor in the sacral region. He tries to avoid an injury of the nerve that arises from the lumbosacral plexus and remains within the abdominal or pelvic cavity. To which of the following nerves should he pay particular attention? (A) Ilioinguinal nerve (B) Genitofemoral nerve (C) Lumbosacral trunk (D) Femoral nerve (E) Lateral femoral cutaneous nerve

(C) The lumbosacral trunk is formed by part of the ventral ramus of the fourth lumbar nerve and the ventral ramus of the fifth lumbar nerve. This trunk contributes to the formation of the sacral plexus by joining the ventral ramus of the first sacral nerve in the pelvic cavity and does not leave the pelvic cavity. All other nerves leave the abdominal and pelvic cavities.

During the initial examination of a 3.6 kg (8 lb) male infant delivered at term, urine is found to be leaking from the umbilicus. This infant most likely has an abnormality of which of the following fetal structures? (A) Umbilical arteries (B) Umbilical vein (C) Urachus (D) Urogenital sinus (E) Urorectal septum

(C) The obliterated urachus is a fetal structure that functions by draining urine from the bladder through the umbilicus into the amniotic sac. If the urachus remains patent, it is possible that urine may be leaking out of the umbilicus.

The ovarian arteries arise from which of the following arteries? (A) superior gluteal (B) inferior gluteal (C) abdominal aorta (D) superior rectal (E) internal pudendal

(C) The ovarian arteries arise from the abdominal aorta.

Which of the following nerves provides parasympathetic innervation to the ovaries? (A) pudendal (B) pelvic splanchnic (C) vagus (D) obturator (E) superior hypogastric

(C) The parasympathetic fibers in the ovarian plexus are derived from the vagus nerve.

The pelvic splanchnic nerves contain which of the following? (A) somatic afferents (B) sympathetic fibers (C) parasympathetic fibers (D) special visceral efferents (E) special somatic afferents

(C) The pelvic splanchnic nerves contain parasympathetic fibers derived from S2, S3, and S4 spinal cord segments and visceral afferent fibers from cell bodies in the spinal ganglia of the corresponding spinal nerves.

All of the following statements concerning the ureters are correct EXCEPT (A) They are retroperitoneal. (B) They cross the pelvic brim anterior to the bifurcation of the common iliac arteries. (C) They are fibrous tubes connecting the kidneys to the urinary bladder. (D) Their superior halves lie in the abdomen and the inferior halves lie in the pelvis. (E) Their oblique passage through the bladder wall forms a one-way "flap valve."

(C) The ureters are muscular tubes, 25 to 30 cm long, that connect the kidneys to the urinary bladder. They are retroperitoneal, with their superior halves in the abdomen and their inferior halves in the pelvis. The pelvic part of the ureters begins where it crosses the bifurcation of the common iliac artery.

All of the following statements concerning the uterus are correct EXCEPT (A) It is anteverted. (B) It is anteflexed. (C) The position of the uterus is fixed. (D) It is divisible into two main parts. (E) The body of the uterus lies between the layers of the broad ligament.

(C) The uterus is a thick-walled, pear-shaped muscular organ whose body lies between the layers of the broad ligament. In the adult, the uterus is usually anteverted and anteflexed; its position changes with the degree of fullness of the bladder and rectum.

A 15-year-old girl delivering her first child at home with only her mother's help experiences a difficult birthing. As the baby's head passes through the birth canal, the perineum begins to tear. Which of the following muscles is the most likely to tear in this event? (A) Coccygeus (B) Ischiocavernosus (C) Obturator internus (D) Pubococcygeus (E) External anal sphincter

(D)

A forensic anthropologist is called to the basement of an abandoned house, where an intact human skeleton has been discovered in a shallow grave. Without the benefit of soft tissue, the gender of the body can still be established based upon the characteristics of the two unearthed coxal (hip) bones. Which of the following characteristics is indicative of the female pelvis? (A) Subpubic angle less than 70 degrees (B) Heart-shaped (android) pelvic inlet (C) More inverted ischial tuberosities (D) More everted alae of the ilia (E) Ischial spines close together in the midline

(D)

A male infant is born with undescended testes that remain intra-abdominal 5 months later (cryptorchidism). Each testis possesses an elongate, distinctive gubernaculum testis running into the scrotum. Which of the following structures in female anatomy is homologous to the gubernaculum testis? (A) Uterine tube (B) Ureter (C) Labium minus (D) Round ligament of the uterus (E) Vestibular bulb

(D)

As part of a complete physical examination, an obstetrician conducts a digital vaginal examination of her 25-year-old patient. Which of the following structures is normally palpable through the posterior fornix? (A) Urethra (B) Ovaries (C) Ischial spines (D) Perineal body (E) Ureters

(D)

Familial dysautonomia is a rare genetic disorder characterized by abnormal functioning of the autonomic nervous system. Problems include difficulty in feeding and respiration, vasomotor instability, insensitivity to pain, and ataxia (an unsteady gait). Patients with this syndrome have low numbers of autonomic (visceral motor) neurons, probably related to defects in production and/or survival of the neural crest precursors of these neurons. Such a disorder could directly affect the innervation of the muscle cells in which of the following structures? (A) Pelvic diaphragm (B) External anal sphincter (C) Urogenital diaphragm (D) Internal urethral sphincter (E) Levator ani

(D)

A woman is delivering a breech baby. The obstetrician decides that it is best to perform a mediolateral episiotomy. Which of the following structures should the obstetrician avoid incising? (A) Vaginal wall (B) Superficial transverse perineal muscle (C) Bulbospongiosus (D) Levator ani (E) Perineal membrane

(D) An obstetrician should avoid incising the levator ani and the external anal sphincter. The levator ani is the major part of the pelvic diaphragm, which forms the pelvic floor and supports all of the pelvic organs. None of the other choices applies here.

An obstetrician performs a median episiotomy on a woman before parturition to prevent uncontrolled tearing. If the perineal body is damaged, the function of which of the following muscles might be impaired? (A) Ischiocavernosus and sphincter urethrae (B) Deep transverse perineal and obturator internus (C) Bulbospongiosus and superficial transverse perineal (D) External anal sphincter and sphincter urethrae (E) Bulbospongiosus and ischiocavernosus

(C) The perineal body (central tendon of the perineum) is a fibromuscular node at the center of the perineum. It provides attachment for the bulbospongiosus, the superficial and deep transverse perineal muscles, and the sphincter ani externus muscles. Other muscles (ischiocavernosus, sphincter urethrae, and obturator internus) are not attached to the perineal body.

A 6 month-old male infant is admitted to the children's hospital because he has no testis in his scrotum. During physical examination, the pediatrician palpated the testis in the inguinal canal. What is the diagnosis of this condition? (A) Male pseudohermaphroditism (B) Hypospadias (C) Epispadias (D) Cryptorchid testis (E) Chordee

(D) Cryptorchid testis is called an undescended testis, which is located in the inguinal region. Male pseudohermaphroditism is a condition in which the affected individual is a genetic and gonadal male with genital anomalies. Hypospadias occurs when the spongy urethra opens on the underside of the penis, frequently associated with the chordee, which is a ventral curvature of the penis. Epispadias occurs when the urethra opens on the dorsal surface of the penis.

Which of the following structures extends into the labia majora? (A) suspensory ligament of the ovary (B) ligament of the ovary proper (C) processus vaginalis (D) round ligament of the uterus (E) uterine tube

(D) Descent of the gonads is considerably less in the female than in the male, and the ovaries finally settle just below the rim of the true pelvis. The cranial genital ligament forms the suspensory ligament of the ovary, whereas the caudal genital ligament forms the ligament of the ovary proper and the round ligament of the uterus. The latter extends into the labia majora.

A 59-year-old man is diagnosed with prostate cancer following a digital rectal examination. For the resection of prostate cancer, it is important to know that the prostatic ducts open into or on which of the following structures? (A) Membranous part of the urethra (B) Seminal colliculus (C) Spongy urethra (D) Prostatic sinus (E) Prostatic utricle

(D) Ducts from the prostate gland open into the prostatic sinus, which is a groove on either side of the urethral crest. The prostate gland receives the ejaculatory duct, which opens into the prostatic urethra on the seminal colliculus (a prominent elevation of the urethral crest) just lateral to the prostatic utricle, which is a small blind pouch. The bulbourethral gland lies on the lateral side of the membranous urethra within the deep perineal space, but its duct opens into the bulbous portion of the spongy (penile) urethra.

The posterior part of the tendinous arch of pelvic fascia forms which of the following ligaments? (A) puboprostatic (B) pubovesicle (C) transverse cervical (D) sacrogenital (E) sacrospinous

(D) The anterior part of the tendinous arch of pelvic fascia forms the puboprostatic ligament in males or the pubovesical ligament in females. The posterior part of the tendinous arch of pelvic fascia forms the sacrogenital ligaments from the sacrum around the side of the rectum to attach to the prostate in the male or the vagina in the female.

The deep arteries of the penis are located in which of the following areas? (A) superficial to the tunica albuginea (B) superficial to the deep fascia (C) within the corpus spongiosum (D) within the corpora cavernosa (E) prepuce

(D) The arterial supply of the penis is mainly from the branches of the internal pudendal arteries. The deep arteries of the penis are the main vessels supplying the cavernous spaces in the erectile tissue of the corpora cavernosa and are therefore involved in the erection of the penis. The deep arteries pierce the crura and run within the corpora cavernosa.

Which structure can be palpated anterior to the cervix during a pelvic examination? (A) Cardinal ligament (B) Ovary (C) Pelvic diaphragm (D) Bladder (E) Uterine tube

(D) The bladder is anterior to the vagina.

An elderly man with prostatitis is seen at an internal medicine clinic. The seminal colliculus of his prostate gland is infected, and its fine openings are closed. Which of the following structures is/are most likely to be disturbed? (A) Ducts of the prostate gland (B) Prostatic utricle (C) Ducts of the bulbourethral glands (D) Ejaculatory ducts (E) Duct of the seminal vesicles

(D) The ejaculatory ducts, which open onto the seminal colliculus, may be injured. The prostate ducts open into the urethral sinus, the bulbourethral ducts open into the bulbous part of the penile urethra, and the ducts of the seminal vesicle join the ampulla of the ductus deferens to form the ejaculatory duct. The prostatic utricle is a minute pouch on the summit of the seminal colliculus.

The female pubic arch differs from the male pubic arch to facilitate childbirth. When compared to the male, the female pubic arch can best be described as (A) narrower (B) shorter (C) taller (D) wider

(D) The female pubic arch is wider than the male pubic arch. The female pubic arch is about 85 degrees compared to 60 degrees in the male.

Which of the following muscles surrounds the crura in the root of the penis? (A) superficial transverse perineal (B) deep transverse perineal (C) bulbospongiosus (D) ischiocavernosus (E) cremaster

(D) The ischiocavernous muscles surround the crura in the root of the penis. Each muscle arises from the internal surface of the ischial tuberosity and ischial ramus and passes anteriorly on the crus of the penis, where it is inserted into the sides and ventral surface of the crus and the perineal membrane. The ischiocavernous muscles force blood from the cavernous spaces in the crura into the distal parts of the corpora cavernosa, thus increasing the turgidity of the penis.

A sexually active adolescent presents with an infection within the ischiorectal fossa. Which of the following structures is most likely injured? (A) Vestibular bulb (B) Seminal vesicle (C) Greater vestibular gland (D) Inferior rectal nerve (E) Internal pudendal artery

(D) The ischiorectal fossa contains the inferior rectal nerves and vessels and adipose tissue. The bulb of the vestibule and the great vestibular gland are located in the superficial perineal space, whereas the bulbourethral gland is found in the deep perineal space. The internal pudendal artery runs in the pudendal canal, but its branches pass through the superficial and deep perineal spaces.

Which of the following areas contributes to the major part of the prostate? (A) anterior lobe (B) isthmus (C) posterior lobe (D) lateral lobes (E) middle lobe

(D) The lateral lobes on either side of the urethra form the major part of the prostate.

The navicular fossa is located in which of the following structures? (A) bulb of the penis (B) prostate (C) membranous urethra (D) glans penis (E) neck of bladder

(D) The lumen of the spongy urethra is approximately 5 mm in diameter: however, it is expanded in the bulb of the penis to form the intrabulbar fossa and in the glans penis to form the navicular fossa.

A 23-year-old massage therapist who specializes in women's health attends a lecture at an annual conference on techniques of massage. She asks, "What structure is drained by the lumbar (aortic) lymph nodes?" Which of the following structures is the correct answer to this question? (A) Perineum (B) Lower part of the vagina (C) External genitalia (D) Ovary (E) Lower part of the anterior abdominal wall

(D) The lymphatic vessels from the ovary ascend with the ovarian vessels in the suspensory ligament and terminate in the lumbar (aortic) nodes. Lymphatic vessels from the perineum, external genitalia, and lower part of the anterior abdominal wall drain into the superficial inguinal nodes.

All of the following statements concerning the pectinate line are correct EXCEPT (A) It is the junction between the superior and inferior parts of the anal canal. (B) The superior rectal artery supplies the superior part of the anal canal. (C) Lymphatic vessels from the superior part of the anal canal drain into the internal lymph nodes. (D) The superior part of the anal canal is drained by the internal rectal venous plexus which drains into tributaries of the caval venous system. (E) The nerve supply to the anal canal superior to the pectinate line is somatic innervation.

(D) The pectinate line indicates the junction of the superior part of the anal canal and the inferior part. The anal canal superior to the pectinate line differs from the part inferior to the pectinate line in its arterial supply, innervation, and venous and lymphatic drainage. The superior rectal artery supplies the superior part of the anal canal and the inferior rectal arteries supply the inferior part. Superior to the pectinate line, the internal rectal venous plexus drains chiefly into the superior rectal vein—a tributary of the inferior mesenteric vein and the portal system. Inferior to the pectinate line the internal rectal plexus drains into the inferior rectal veins—tributaries to the caval venous system. Superior to the pectinate line, the lymphatics drain into the internal iliac lymph nodes into the common iliac and lumbar nodes. Inferior to the pectinate line, the lymphatics drain into the superficial inguinal lymph nodes. Superior to the pectinate line, the innervation is visceral; inferior to the pectinate line, the innervation is somatic.

A general surgeon is giving a lecture to a team of surgery residents. She describes characteristics of structures above the pectinate line of the anal canal, which include: (A) Stratified squamous epithelium (B) Venous drainage into the caval system (C) Lymphatic drainage into the superficial inguinal nodes (D) Visceral sensory innervation (E) External hemorrhoids

(D) The pectinate line is a point of demarcation between visceral and somatic portions of the anal canal. Characteristics above the pectinate line include columnar epithelium, venous drainage into the portal system, lymphatic drainage into the internal iliac nodes, visceral sensory innervation, and internal hemorrhoids.

A 56-year-old man who is diagnosed with rectal cancer is undergoing biopsy of several lymph nodes. The nodes most likely to be sampled from this patient will be from the inferior mesenteric nodes, inguinal nodes, and the (A) gonadal nodes (B) internal iliac nodes (C) portal vein nodes (D) renal nodes (E) superior mesenteric nodes

(B) Lymphatics in the abdomen generally follow its associated arteries. Clusters of lymph nodes, which are important in monitoring the immune system, are found along the course of the regional arteries. The rectum is supplied by the following: ■ Superior rectal artery—branch off the inferior mesenteric artery ■ Middle rectal artery—branch off the internal iliac artery ■ Inferior rectal artery—branch off the internal pudendal artery Therefore, if the rectal cancer spreads, it can potentially do so parallel to all three arterial origins. The lymph nodes for the superior and inferior rectal arteries are provided in the stem of the question. The only nodes not mentioned are the internal iliac nodes for the origin of the middle rectal artery.

A 48-year-old college football coach undergoes a radical prostatectomy for a malignant tumor in his prostate. Following surgery, he is incapable of achieving an erection. Which of the following nerves is most likely damaged during the surgery? (A) Sacral splanchnic nerve (B) Pelvic splanchnic nerve (C) Pudendal nerve (D) Dorsal nerve of the penis (E) Posterior scrotal nerve

(B) Parasympathetic preganglionic fibers in the pelvic splanchnic nerve are responsible for erection of the penis. Sympathetic preganglionic fibers in the sacral splanchnic nerve are responsible for ejaculation. The pudendal nerve supplies the external anal sphincter and perineal muscles and supplies GSA fi bers to the perineal region. The dorsal nerve of the penis is a terminal branch of the pudendal nerve and supplies sensation of the penis. The posterior scrotal nerves are superficial branches of the perineal nerve and supply sensory fibers to the scrotum.

Some obstetricians apply the term perineum to a more restricted region that extends between which of the following structures? (A) perineal body and the mons pubis (B) vagina and anus (C) pubic arch and the rectum (D) ischial spines and pubic tubercles (E) vestibule and sacrum

(B) Some obstetricians apply the term perineum to a more restricted region that includes the area between the vagina and anus.

A 26-year-old man comes to a hospital with fever, nausea, pain, and itching in the perineal region. On examination by a urologist, he is diagnosed as having infected bulbourethral (Cowper's) glands. Which of the following structures is/are affected by this infection? (A) Superficial perineal space (B) Sphincter urethrae (C) Production of sperm (D) Testis (E) Seminal vesicles

(B) The bulbourethral glands lie on either side of the membranous urethra, embedded in the sphincter urethrae. Their ducts open into the bulbous part of the penile urethra. Semen—a thick, yellowish-white, viscous, spermatozoa-containing fluid—is a mixture of the secretions of the testes, seminal vesicles, prostate, and bulbourethral glands. Sperm, or spermatozoa, are produced in the seminiferous tubules of the testis and mature in the head of the epididymis. The seminal vesicles are lobulated glandular structures, produce the alkaline constituent of the seminal fluid that contains fructose and choline, and lie inferior and lateral to the ampullae of the ductus deferens against the fundus (base) of the bladder.

After his bath but before getting dressed, a 4-year-old boy was playing with his puppy. The boy's penis was bitten by the puppy, and the deep dorsal vein was injured. The damaged vein: (A) Lies superficial to Buck's fascia (B) Drains into the prostatic venous plexus (C) Lies lateral to the dorsal artery of the penis (D) Is found in the corpus spongiosum (E) Is dilated during erection

(B) The deep dorsal vein of the penis lies medial to the dorsal artery of the penis on the dorsum of the penis and deep to Buck's fascia, drains into the prostatic plexus of veins, and is compressed against the underlying deep fascia of the penis during erection.

A 43-year-old man has a benign tumor located near a gap between the arcuate pubic ligament and the transverse perineal ligament. Which of the following structures is most likely compressed by this tumor? (A) Perineal nerve (B) Deep dorsal vein of the penis (C) Superficial dorsal vein (D) Posterior scrotal nerve (E) Deep artery of the penis

(B) The deep dorsal vein, dorsal artery, and dorsal nerve of the penis pass through a gap between the arcuate pubic ligament and the transverse perineal ligament. The perineal nerve divides into a deep branch, which supplies all of the perineal muscles, and superficial branches as posterior scrotal nerves that supply the scrotum. The superficial dorsal vein of the penis empties into the greater saphenous vein. The deep artery of the penis runs in the corpus cavernosum of the penis.

When performing gastric bypass surgery on a 36-year-old woman, the surgeon identifies the hepatogastric and hepatoduodenal ligaments. Together, both ligaments create which of the following structures? (A) Greater omentum (B) Lesser omentum (C) Mesentery (D) Parietal peritoneum (E) Omental bursa (F) Visceral peritoneum

(B) The hepatogastric and hepatoduodenal ligaments are the two components of the lesser omentum. They are named for their attachments to the liver, stomach, and duodenum.

A 32-year-old man undergoes vasectomy as a means of permanent birth control. A physician performing the vasectomy by making an incision on each side of the scrotum should remember which of the following statements most applicable to the scrotum? (A) It is innervated by the ilioinguinal and genitofemoral nerves. (B) It receives blood primarily from the testicular artery. (C) Its venous blood drains primarily into the renal vein on the left. (D) Its lymphatic drainage is primarily into upper lumbar nodes. (E) Its dartos tunic is continuous with the perineal membrane.

(A) The scrotum is innervated by branches of the ilioinguinal, genitofemoral, pudendal, and posterior femoral cutaneous nerves. The scrotum receives blood from the posterior scrotal branches of the internal pudendal arteries and the anterior scrotal branches of the external pudendal arteries, but it does not receive blood from the testicular artery. Similarly, the scrotum is drained by the posterior scrotal veins into the internal pudendal vein. The lymph vessels from the scrotum drain into the superficial inguinal nodes, whereas the lymph vessels from the testis drain into the upper lumbar nodes. The dartos tunic is continuous with the membranous layer of the superficial perineal fascia (Colles's fascia).

The lesser vestibular glands open into which of the following structures? (A) vestibule between the urethral and vaginal orifices (B) vestibule on each side of the vaginal orifice (C) bulbs of the vestibule (D) glans clitoris (E) bulbourethral ducts

(A) The slender ducts of the greater vestibular glands pass deep to the bulbs of the vestibule and open into the vestibule on each side of the vaginal orifice. The lesser vestibular glands are small glands on each side of the vestibule that open into it between the urethral and vaginal orifices.

Which of the following arteries is a direct continuation of the inferior mesenteric artery? (A) superior rectal (B) middle rectal (C) inferior rectal (D) iliolumbar (E) lateral sacral

(A) The superior rectal artery is the direct continuation of the inferior mesenteric artery. The superior rectal artery anastomoses with branches of the middle rectal artery (a branch of the internal iliac artery) and with the inferior rectal artery (a branch of the internal pudendal artery).

A 42-year-old woman who has had six children develops a weakness of the urogenital diaphragm. Paralysis of which of the following muscles would cause such a symptom? (A) Sphincter urethrae (B) Coccygeus (C) Superficial transversus perinei (D) Levator ani (E) Obturator internus

(A) The urogenital diaphragm consists of the sphincter urethrae and deep transverse perineal muscles. Weakness of the muscles, ligaments, and fasciae of the pelvic fl oor, such as the pelvic diaphragm, urogenital diaphragm, and cardinal (transverse cervical) ligaments, occurs as result of multiple child delivery, advancing age, and menopause. The pelvic diaphragm is composed of the levator ani and coccygeus muscles. The superficial transversus perinei is one of the superficial perineal muscles, and the obturator internus forms the lateral wall of the ischiorectal fossa.

The vaginal fornices are derived from which of the following structures? (A) paramesonephric duct (B) mesonephric ducts (C) ectodermal duct (D) sinovaginal bulbs (E) urogenital sinus

(A) The winglike expansions of the vagina around the end of the uterus, the vaginal fornices, are of paramesonephric origin.

Weak areas of the pelvis include all of the following EXCEPT (A) ischial tuberosities (B) alae of the ilium (C) pubic rami (D) sacroiliac joint (E) acetabula

(A) Weak areas of the pelvis include the pubic rami, acetabula, sacroiliac joint, and alae of the ilium.

A 20-year-old woman delivers a stillborn infant with bilateral agenesis of the kidneys. During the later stages of pregnancy, the fetus likely also had which of the following conditions? (A) Polyhydramnios (B) Oligohydramnios (C) Renal hypoplasia (D) Pelvic kidneys (E) Polycystic kidneys

(B)

A 65-year-old woman suffers chronic constipation. Which of the following events occurs when she strains heavily in attempting to pass her stool? (A) Relaxation of the pubococcygeus (B) Relaxation of the puborectalis (C) Relaxation of the coccygeus (D) Contraction of the external anal sphincter (E) Contraction of the internal anal sphincter

(B)

A 70-year-old patient with a history of hypercholesterolemia complains of severe cramps in his gluteal region despite no history of physical exertion in the past few days. An arteriogram shows atherosclerotic blockage leading to insufficient perfusion to the gluteal region. What artery is most likely occluded leading to his complications? (A) External iliac (B) Internal iliac (C) Femoral (D) Internal pudendal (E) Abdominal aorta

(B)

A medical resident is charged with catheterizing the urethra in a 68-year-old man in order to drain urine from the bladder. Which of the following sequence of structures correctly lists the order of the structures encountered when passing a catheter through the external urethral orifice to the urinary bladder? (A) Navicular fossa, spongy urethra, membranous urethra, ductus deferens, prostatic urethra (B) Navicular fossa, spongy urethra, membranous urethra, prostatic urethra, intramural urethra (C) Spongy urethra, membranous urethra, prostatic urethra, intramural urethra, ureter (D) Spongy urethra, membranous urethra, ejaculatory duct, ductus deferens, prostatic urethra (E) Intramural urethra, prostatic urethra, membranous urethra, spongy urethra, navicular fossa

(B)

As part of a general pelvic examination, an obstetrician measures the diagonal conjugate diameter of the true pelvis in his 26-year-old pregnant patient. This measurement is taken between which of the following points? (A) Ischial spine to the opposite ischial spine (B) Sacral promontory to the inferior edge of the pubic symphysis (C) Sacral promontory to the posterosuperior edge of the pubic symphysis (D) Sacroiliac joint to the posterior side of the opposite side body of the pubis (E) Tip of the coccyx to the pubic symphysis

(B)

During dissection of deep pelvic lymph nodes in a 60- year-old man, the surgeon accidentally severs the internal pudendal artery at its origin. Which of the following structures will maintain its normal blood supply immediately following this lesion? (A) Penis (B) Urinary bladder (C) Anal canal (D) Scrotum (E) Urogenital diaphragm

(B)

During micturition (urination), excitation of parasympathetic nerve fibers causes contraction of which of the following muscles? (A) Pubococcygeus muscle (B) Detrusor muscle (C) Internal urethral sphincter (D) Bulbospongiosus muscle (E) External urethral sphincter

(B)

Multiple tumors in the pelvic cavity cause widespread destruction through the hypogastric plexuses in a 67-year-old man. The nerve fibers that degenerate as a result of these lesions are mainly those of which of the following? (A) Presynaptic sympathetic neurons (B) Presynaptic parasympathetic neurons (C) Postsynaptic parasympathetic neurons (D) Somatic motor neurons (E) General sensory neurons

(B)

A 29-year-old woman with a ruptured ectopic pregnancy is admitted to a hospital for culdocentesis. A long needle on the syringe is most efficiently inserted through which of the following structures? (A) Anterior fornix of the vagina (B) Posterior fornix of the vagina (C) Anterior wall of the rectum (D) Posterior wall of the uterine body (E) Posterior wall of the bladder

(B) A needle should be inserted through the posterior fornix just below the posterior lip of the cervix while the patient is in the supine position to aspirate abnormal fl uid in the cul-de-sac of Douglas (rectouterine pouch). Rectouterine excavation is not most efficiently aspirated by puncture of other structures.

A 32-year-old patient with multiple fractures of the pelvis has no cutaneous sensation in the urogenital triangle. The function of which of the following nerves is most likely to be spared? (A) Ilioinguinal nerve (B) Iliohypogastric nerve (C) Posterior cutaneous nerve of the thigh (D) Pudendal nerve (E) Genitofemoral nerve

(B) The iliohypogastric nerve innervates the skin above the pubis. The skin of the urogenital triangle is innervated by the pudendal nerve, perineal branches of the posterior femoral cutaneous nerve, anterior scrotal or labial branches of the ilioinguinal nerve, and the genital branch of the genitofemoral nerve.

The internal iliac artery is crossed by which of the following structures at the level of the 4th intervertebral disc between L5 and S1? (A) puborectalis muscle (B) ureter (C) obturator nerve (D) umbilical artery (E) tendinous arch of the levator ani

(B) The internal iliac artery begins at the level of the fourth disc between L5 and S1 vertebrae, where it is crossed by the ureter. It is separated from the sacroiliac joint by the internal iliac vein and the lumbosacral trunk.

A 17-year-old boy suffers a traumatic groin injury during a soccer match. The urologist notices tenderness and swelling of the boy's left testicle that may be produced by thrombosis in which of the following veins? (A) Left internal pudendal vein (B) Left renal vein (C) Inferior vena cava (D) Left inferior epigastric vein (E) Left external pudendal vein

(B) A tender swollen left testis may be produced by thrombosis in the left renal vein because the left testicular vein drains into the left renal vein. The right testicular vein drains into the inferior vena cava. The left internal pudendal vein empties into the left internal iliac vein. The left inferior epigastric vein drains into the left external iliac vein, and the left external pudendal vein empties into the femoral vein.

A trauma surgeon in the emergency department at a local center examines a 14-year-old boy with extensive pelvic injuries after a hit and run accident. The surgeon inspects the ischiorectal fossa because it: (A) Accumulates urine leaking from rupture of the bulb of the penis (B) Contains the inferior rectal vessels (C) Has a pudendal canal along its medial wall (D) Is bounded anteriorly by the sacrotuberous ligament (E) Contains a perineal branch of the fifth lumbar nerve

(B) The ischiorectal fossa is bounded posteriorly by the gluteus maximus and the sacrotuberous ligament. It contains fat, the inferior rectal nerve and vessels, and perineal branches of the posterior femoral cutaneous nerve. The pudendal canal runs along its lateral wall. Urine leaking from a ruptured bulb of the penis does not spread into the ischiorectal fossa because Scarpa's fascia ends by firm attachment to the fascia lata of the thigh.

A 70-year-old-man has a blockage at the origin of the inferior mesenteric artery. He does not have ischemic pain because of collateral arterial supply.Which of the following arteries is the most likely additional source of blood to the descending colon? (A) Left gastroepiploic (B) Middle colic (C) Sigmoid (D) Splenic (E) Superior rectal

(B) The marginal artery of Drummond consists of contributions from the inferior mesenteric artery as well as branches from the superior mesenteric artery via the right and middle colic arteries. Therefore, if the inferior mesenteric artery is blocked, blood flowing from the middle colic artery would provide the additional source of blood to the descending colon.

An elderly man with benign enlargement of his prostate experiences difficulty in urination, urinary frequency, and urgency. Which of the following lobes of the prostate gland is commonly involved in benign hypertrophy that obstructs the prostatic urethra? (A) Anterior lobe (B) Middle lobe (C) Right lateral lobe (D) Left lateral lobe (E) Posterior lobe

(B) The middle lobe of the prostate gland is commonly involved in benign prostatic hypertrophy, resulting in obstruction of the prostatic urethra, whereas the posterior lobe is commonly involved in carcinomatous transformation. The anterior lobe contains little glandular tissue, and the two lateral lobes on either side of the urethra form the major part of the gland.

The posterior fornix is the deepest part of which of the following structures? (A) urinary bladder (B) vagina (C) deep perineal space (D) urethra (E) uterus

(B) The posterior fornix is the deepest part of the vagina and is closely related to the rectouterine pouch. The vaginal fornix is the recess around the cervix and is described as having anterior, posterior, and lateral parts.

Which of the following parts of the male urethra is the widest and most dilatable? (A) preprostatic (B) prostatic (C) membranous (D) spongy (E) external urethral meatus

(B) The prostatic urethra is the widest and most dilatable part of the male urethra.

The rounded vaginal part of the cervix extends into the vagina and communicates with it through which of the following structures? (A) uterine tubes (B) external os (C) ureter (D) urethra (E) internal os

(B) The rounded vaginal part of the cervix extends into the vagina and communicates with it through the external os. The cervical canal is broadest at its middle part and communicates with the uterine cavity through the internal os and with the vagina through the external os.

A 21-year-old man is involved in a highspeed motor vehicle accident. As a result, he has extensive damage to his sphincter urethrae. Which of the following best describes the injured sphincter urethrae? (A) Smooth muscle (B) Innervated by the perineal nerve (C) Lying between the perineal membrane and Colles's fascia (D) Enclosed in the pelvic fascia (E) Part of the pelvic diaphragm

(B) The sphincter urethrae is striated muscle that lies in the deep perineal space and forms a part of the urogenital diaphragm but not the pelvic diaphragm. It is not enclosed in the pelvic fascia. It is innervated by a deep (muscular) branch of the perineal nerve.

The superior gluteal artery leaves the pelvis through which of the following openings? (A) greater sciatic foramen (B) lesser sciatic foramen (C) obturator canal (D) pudendal canal (E) sacral foramina

(B) The superior gluteal artery leaves the pelvis through the superior part of the greater sciatic foramen, superior to the piriformis muscle, to supply the gluteal muscles in the buttocks.

All of the following statements concerning the sciatic nerve are correct EXCEPT (A) It is the largest and broadest nerve in the body. (B) It is formed by the dorsal rami of L4 to S3. (C) It passes through the greater sciatic foramen. (D) It exits the pelvis inferior to the piriformis muscle. (E) It is one of the two main nerves of the sacral plexus.

(B) The two main nerves of the sacral plexus include the sciatic and pudendal. The sciatic nerve is the largest and broadest nerve in the body. It is formed by the ventral rami of L4-S3, which converge on the anterior surface of the piriformis. The sciatic nerve usually passes through the greater sciatic foramen, inferior to the piriformis, to enter the gluteal region.

The uvula of the bladder is located in which of the following locations? (A) retropubic space (B) trigone of the bladder (C) apex of the bladder (D) fundus of the bladder (E) pubovesical ligament

(B) The uvula of the bladder is a slight projection of the trigone of the bladder. It is usually more prominent in older men.

The paraurethral glands are homologues to which of the following structures? (A) testes (B) prostate (C) seminal vesicles (D) bulbourethral glands (E) epididymis

(B) Urethral glands are present particularly in the superior part of the female urethra. The paraurethral glands are homologues to the prostate.

A 26-year-old medical student noticed a lump on his left testis, which was later diagnosed as testicular cancer. To rule out metastases of the testicular cancer, what group of lymph nodes should be checked in the fi rst instance? (A) Superfi cial inguinal (B) Deep inguinal (C) Lumbar (D) External iliac (E) Internal iliac

(C)

A 72-year-old man comes to his urologist complaining of urinary frequency, urgency, weak urinary stream, hesitancy, and straining to void. A rectal examination and blood tests revealing normal prostate-specifi c antigen levels lead to a diagnosis of benign prostatic hyperplasia. The man undergoes a prostatectomy to remove the prostate gland. During the surgical procedure, the prostatic nerve plexus is damaged. What is the most likely complication of this surgery? (A) Loss of sensation from the anus (B) Fecal incontinence (C) Erectile dysfunction (D) Loss of sensation from the posterior scrotum (E) Vasomotor dysfunction in the rectum

(C)

A 29-year-old woman is admitted to a hospital because the birth of her child is several days overdue. Tearing of the pelvic diaphragm during childbirth leads to paralysis of which of the following muscles? (A) Piriformis (B) Sphincter urethrae (C) Obturator internus (D) Levator ani (E) Sphincter ani externus

(D) The pelvic diaphragm is formed by the levator ani and coccygeus, whereas the urogenital diaphragm consists of the sphincter urethrae and deep transverse perinei muscles. The piriformis passes through the greater sciatic notch and inserts on the greater trochanter of the femur. The obturator internus forms the lateral wall of the ischiorectal fossa. The sphincter ani externus is composed of three layers, including the subcutaneous (corrugator cutis ani), superficial, and deep portions, and maintains a voluntary tonic contracture.

Which of the following structures form a U-shaped sling around the anorectal junction? (A) sacrospinous ligament (B) anococcygeal ligament (C) superficial transverse perineal muscle (D) puborectalis muscle (E) sacrotuberous ligament

(D) The puborectalis muscle unites with its partner to form a U-shaped sling around the anorectal junction. The puborectalis is responsible for the anorectal angle (perineal flexure), which is important in maintaining fecal continence.

A 39-year-old man is unable to expel the last drops of urine from the urethra at the end of micturition because of paralysis of the external urethral sphincter and bulbospongiosus muscles. This condition may occur as a result of injury to which of the following nervous structures? (A) Pelvic plexus (B) Prostatic plexus (C) Pudendal nerve (D) Pelvic splanchnic nerve (E) Sacral splanchnic nerve

(C) The perineal branch of the pudendal nerve supplies the external urethral sphincter and bulbospongiosus muscles in the male. All other nervous structures do not supply skeletal muscles but supply smooth muscles in the perineal and pelvic organs. The pelvic and prostatic plexuses contain both sympathetic and parasympathetic nerve fibers. The pelvic splanchnic nerve carries preganglionic parasympathetic fibers, whereas the sacral splanchnic nerve transmits preganglionic sympathetic fibers.

The pelvic floor is formed by all of the following muscles EXCEPT (A) pubococcygeus (B) coccygeus (C) piriformis (D) puborectalis (E) iliococcygeus

(C) The piriformis muscles cover the posterolateral wall of the pelvis. The pelvic floor is formed by the funnel-shaped pelvic diaphragm, which consists of the levator ani and coccygeus. The levator ani includes the pubococcygeus, puborectalis, and iliococcygeus.

Which of the following structures is the largest accessory gland of the male reproductive system? (A) testes (B) seminal vesicles (C) prostate (D) bulbourethral glands (E) epididymis

(C) The prostate is the largest accessory gland of the male reproductive system.

An obstetrician is about to perform a pudendal block so a woman can experience less pain when she delivers her child. He recalls what he learned in medical school about this nerve. Which of the following statements is correct? (A) It passes superficial to the sacrotuberous ligament. (B) It innervates the testis and epididymis in a male. (C) It provides motor fibers to the coccygeus. (D) It can be blocked by injecting an anesthetic near the inferior margin of the ischial spine. (E) It arises from the lumbar plexus.

(D) The pudendal nerve, which arises from the sacral plexus, provides sensory innervation to the labium majus (or scrotum in a male). It leaves the pelvis through the greater sciatic foramen and enters the perineum through the lesser sciatic foramen near the inferior margin of the ischial spine. Therefore, it can be blocked by injection of an anesthetic near the inferior margin of the ischial spine.

Rapid elongation of the genital tubercle in the male gives rise to which of the following structures? (A) testes (B) scrotum (C) ductus deferens (D) phallus (E) epididymis

(D) The rapid elongation of the genital tubercle in the male gives rise to the phallus.

A 22-year-old woman receives a deep cut in the inguinal canal 1 in. lateral to the pubic tubercle. Which of the following ligaments is lacerated within the inguinal canal? (A) Suspensory ligament of the ovary (B) Ovarian ligament (C) Mesosalpinx (D) Round ligament of the uterus (E) Rectouterine ligament

(D) The round ligament of the uterus is found in the inguinal canal along its course. The other ligaments do not pass through the inguinal canal.

Which of the following structures most likely converts the greater sciatic notch to the greater sciatic foramen? (A) Obturator membrane (B) Obturator internus muscle (C) Piriformis muscle (D) Sacrospinous ligament (E) Sacrotuberous ligament

(D) The sacrospinous ligament courses from the sacrum to the ischial spine and encloses the greater sciatic notch to form a foramen. The obturator membrane and the internus muscle cover the obturator foramen. The piriformis muscle courses through the greater sciatic foramen, but does not form it.

A 22-year-old victim of an automobile accident has received destructive damage to structures that form the boundary of the perineum. Which of the following structures is spared? (A) Pubic arcuate ligament (B) Tip of the coccyx (C) Ischial tuberosities (D) Sacrospinous ligament (E) Sacrotuberous ligament

(D) The sacrospinous ligament forms a boundary of the lesser sciatic foramen. The pubic arcuate ligament, tip of the coccyx, ischial tuberosities, and sacrotuberous ligament all form part of the boundary of the perineum.

A 38-year-old man with a history of "heartburn" suddenly experiences excruciating pain in the epigastric region of his abdomen. Surgery is performed immediately, and evidence of a perforated ulcer in the posterior wall of the stomach is noted. Stomach contents that have seeped out will most likely be found in which of the following structures? (A) Between the parietal peritoneum and the posterior body wall (B) Greater peritoneal sac (C) Ischioanal fossa (D) Lesser peritoneal sac (E) Paracolic gutter

(D) The ulcer in this patient is located on the deep surface of the stomach. Therefore, gastric contents that have seeped out will most likely be found in the lesser peritoneal sac. Recall how the greater peritoneal sac occupies the entire peritoneal cavity, with the exception of the region deep to stomach that is accessed via the epiploic foramen.

A 59-year-old woman comes to a local hospital for uterine cancer surgery. As the uterine artery passes from the internal iliac artery to the uterus, it crosses superior to which of the following structures that is sometimes mistakenly ligated during such surgery? (A) Ovarian artery (B) Ovarian ligament (C) Uterine tube (D) Ureter (E) Round ligament of the uterus

(D) The ureter runs under the uterine artery near the cervix; thus, the ureter is sometimes mistakenly ligated during pelvic surgery. The other structures mentioned are not closely related to the uterine artery near the uterine cervix.

Which of the following structures give rise to the labia minora? (A) mesonephric ducts (B) urogenital groove (C) genital swellings (D) urethral folds (E) genital tubercle

(D) The urethral folds do not fuse, as in the male, but develop into the labia minora.

During the radical hysterectomy of a 52-year-old woman, the surgeon is careful to avoid damaging the ureters when removing the uterus. The landmark relationship that the surgeon should look for adjacent to the uterus to ensure preservation of each ureter is the ureter coursing (A) inferior to the ovarian artery (B) superior to the ovarian artery (C) inferior to the uterine artery (D) superior to the uterine artery (E) inferior to the uterine tube (F) superior to the uterine tube

(D) The uterine artery courses superiorly over the ureter; in other words, "the water (ureter) courses under the bridge (uterine artery)."

A 53-year-old bank teller is admitted to a local hospital for surgical removal of a benign pelvic tumor confined within the broad ligament. There is a risk of injuring which of the following structures that lies in this ligament? (A) Ovary (B) Proximal part of the pelvic ureter (C) Terminal part of the round ligament of the uterus (D) Uterine tube (E) Suspensory ligament of the ovary

(D) The uterine tubes lie in the broad ligament. The anterior surface of the ovary is attached to the posterior surface of the broad ligament of the uterus. The ureter descends retroperitoneally on the lateral pelvic wall but is crossed by the uterine artery in the base (in the inferomedial part) of the broad ligament. The terminal part of the round ligament of the uterus becomes lost in the subcutaneous tissue of the labium majus. The suspensory ligament of the ovary is a band of peritoneum that extends superiorly from the ovary to the pelvic wall.

A 16-year-old boy, who was riding his skateboard down a stair railing, falls and impacts his perineum on the railing, causing a straddle injury. He presents in the ER with discoloration and swelling in the scrotum, penis, and anterior abdominal wall, which is characteristic of extravasation of urine. Which of the following structures is most likely compromised? (A) Ureter (B) Urinary bladder (C) Prostatic urethra (D) Intermediate urethra (E) Spongy urethra

(E)

A 72-year-old woman, who lost 50 lb (22.7 kg) of weight in the past year, visited her physician complaining of abdominal cramps, pain, and vomiting. Physical examination detected a distended abdomen with no visible or palpable masses in the groin and upper thigh area. She also described a sharp cutting pain in the medial aspect of her left thigh, which was exacerbated by the physician extending and medially rotating her left thigh at the hip joint. An abdominal CT revealed small bowel obstruction. What is the most likely cause of the obstruction? (A) Direct inguinal hernia (B) Indirect inguinal hernia (C) Umbilical hernia (D) Femoral hernia (E) Obturator hernia

(E)

A genetic coding error during the 5th week of development causes a unilateral failure of development of the ureteric bud. This condition would directly affect the formation of which of the following structures? (A) Proximal convoluted tubules (B) Renal glomeruli (C) Urinary bladder (D) Uterine tube (E) Ureter

(E)

A pregnant woman in active labor receives an epidural anesthetic to relieve pain from her uterine contractions and cervical dilation in preparation for the birth of her child. Visceral sensory fibers project the pain derived from the dilation of the uterine cervix to what spinal cord level? (A) T4 (B) L1 (C) L4 (D) S1 (E) S3

(E)

Development of the primitive gonads into testes or ovaries is induced by invasion of the genital ridges by the primordial germ cells. These cells migrate from which of the following locations? (A) Paraxial mesoderm (B) Intermediate mesoderm (C) Lateral plate mesoderm (D) Neural crest (E) Yolk sac endoderm

(E)

A 49-year-old woman has a large mass on the pelvic brim. Which of the following structures is most likely compressed by this mass when crossing the pelvic brim? (A) Deep dorsal vein of the penis (B) Uterine tube (C) Ovarian ligament (D) Uterine artery (E) Lumbosacral trunk

(E) All of the listed structures do not cross the pelvic brim except the lumbosacral trunk, which arises from L4 and L5, enters the true pelvis by crossing the pelvic brim, and contributes to the formation of the sacral plexus. The deep dorsal vein of the penis enters the pelvic cavity by passing under the symphysis pubis between the arcuate and transverse perineal ligaments.

Diagnosis of an indirect inguinal hernia is determined when intestine protrudes lateral to the inferior epigastric artery through the abdominal body wall. During the physical examination of a male patient, a physician will assess for an indirect hernia by inserting a finger in the scrotum and feeling for bowel that protrudes, as the patient is instructed to turn his head and cough. If an indirect inguinal hernia is present, the physician will most likely feel bowel at which of the following sites? (A) Anterior superior iliac spine (B) Deep inguinal ring (C) McBurney's point (D) Pubic symphysis (E) Superficial inguinal ring

(E) An indirect hernia results from bowel protruding through the deep inguinal ring, and through the inguinal canal and into the spermatic cord via the superficial inguinal ring. Therefore, during the physical examination, the physician will attempt to feel for herniation by digitally palpating the superficial inguinal ring through the scrotal sac.

All of the following statements concerning the seminal vesicles are correct EXCEPT (A) The peritoneum of the rectovesical pouch separates the superior ends of the seminal vesicles from the rectum. (B) They do not store sperm. (C) They are elongated structures that lie between the fundus of the bladder and the rectum. (D) The inferior ends of the seminal vesicles are separated from the rectum by the rectovesical septum. (E) Bulbourethral glands empty into the seminal vesicles.

(E) Ducts of the bulbourethral glands empty into the urethra. The seminal vesicles are elongated structures that lie between the fundus of the bladder and the rectum, and they do not store sperm. They secrete a thick, alkaline fluid that mixes with the sperm as they pass into the ejaculatory ducts to the urethra. The superior ends of the seminal vesicles are covered with peritoneum and lie posterior to the ureters, where the rectovesical pouch separates them from the rectum. The inferior ends of the seminal vesicle are separated from the rectum by the rectovesical septum.

A young couple is having difficulty conceiving a child. Their physician at a reproduction and fertility clinic explains to them that: (A) The ovary lies within the broad ligament (B) The glans clitoris is formed from the corpus spongiosum (C) Erection of the penis is a sympathetic response (D) Ejaculation follows parasympathetic stimulation (E) Fertilization occurs in the infundibulum or ampulla of the uterine tube

(E) Fertilization takes place in the infundibulum or ampulla of the uterine tube. The glans clitoris is derived from the corpora cavernosa, whereas the glans penis is the expanded terminal part of the corpus spongiosum. Erection of the penis is caused by parasympathetic stimulation, whereas ejaculation is mediated via the sympathetic nerve. The ovaries are not enclosed in the broad ligament, but their anterior surface is attached to the posterior surface of the broad ligament.

All of the following arteries enter the true pelvis EXCEPT (A) internal iliac (B) median sacral (C) superior rectal (D) ovarian (E) testicular

(E) Four main arteries enter the lesser or true pelvis. The internal iliac and ovarian arteries are paired, and the median sacral and superior rectal arteries are unpaired. The testicular artery does not enter the true pelvis, as it follows the testes through the inguinal canal into the scrotum

Immediately superior to the perineal membrane is located which of the following muscles? (A) obturator internus (B) levator ani (C) bulbocavernosus (D) ischiocavernous (E) deep transverse perineal

(E) Immediately superior to the perineal membrane is the deep transverse perineal muscle.

A 23-year-old woman visits her obstetrician for an annual checkup. During vaginal examination, which of the following structures may be palpated? (A) Apex of the urinary bladder (B) Fundus of the uterus (C) Terminal part of the round ligament of the uterus (D) Body of the clitoris (E) Uterine cervix

(E) In addition to the uterine cervix, the uterus, uterine tubes, ovaries, and ureters can be palpated. The apex of the urinary bladder is the anterior end of the bladder; thus, it cannot be palpated. The fundus of the uterus is the anterosuperior part of the uterus. The terminal part of the round ligament of the uterus emerges from the superficial inguinal ring and becomes lost in the subcutaneous tissue of the labium majus.

In females, the deep perineal pouch contains which of the following structures? (A) clitoris (B) greater vestibular glands (C) bulbourethral glands (D) bulbs of the vestibule (E) external urethral sphincter

(E) In females, the deep perineal pouch contains the proximal part of the urethra, the external urethral sphincter muscle, the deep transverse perineal muscles, and related vessels and nerves.

The clitoris is derived from which of the following structures? (A) urethral folds (B) genital swellings (C) urogenital groove (D) mesonephric ducts (E) genital tubercle

(E) In females, the genital tubercle elongates only slightly and forms the clitoris.

Which of the following parts of the uterus protrudes into the uppermost vagina? (A) round ligament (B) body (C) fundus (D) isthmus (E) cervix

(E) Only the cylindrical, narrow inferior part of the uterus known as the cervix protrudes into the uppermost vagina.

Parasympathetic innervation to the hindgut originates in the S2-S4 spinal cord segments. Parasympathetic neurons travel to the prevertebral plexus via which of the following nerves? (A) Greater splanchnic nerves (B) Least splanchnic nerves (C) Lesser splanchnic nerves (D) Lumbar splanchnic nerves (E) Pelvic splanchnic nerves (F) Sacral splanchnic nerves

(E) Pelvic splanchnic nerves exit the ventral rami of spinal nerves S2-S4 and contain preganglionic parasympathetic neurons to the prevertebral plexus, such as the inferior hypogastric plexus. The other splanchnic nerves listed in the choices (i.e., greater, least, lesser, lumbar, and sacral splanchnic nerves) contain only sympathetic neurons.

Which of the following nerves do NOT innervate the scrotum? (A) ilioinguinal (B) genitofemoral (C) pudendal (D) posterior femoral cutaneous (E) obturator

(E) The anterior aspect of the scrotum is supplied by anterior scrotal nerves derived from the ilioinguinal nerve and by the genital branch of the genitofemoral nerve. The posterior aspect of the scrotum is supplied by posterior scrotal nerves, branches of the superficial perineal nerves, and the perineal branch of the posterior femoral cutaneous nerve.

The neck of the bladder in females is held firmly by which of the following structures? (A) puboprostatic ligaments (B) puborectalis muscle (C) levator ani muscle (D) coccygeus muscle (E) pubovesical ligaments

(E) The bladder is relatively free within the extraperitoneal subcutaneous fatty tissue except for its neck, which is held firmly by the puboprostatic ligaments in males and the pubovesical ligaments in females.

Which of the following structures provides support for the ampulla of the rectum? (A) urogenital diaphragm (B) puboprostatic ligament (C) sacrogenital ligament (D) tendinous arch of pelvic fascia (E) levator ani and anococcygeal ligament

(E) The dilated terminal part of the rectum, lying directly above and supported by the pelvic diaphragm (levator ani) and anococcygeal ligament is the ampulla of the rectum.

In males, the superficial perineal pouch contains which of the following structures? (A) prostate (B) seminal vesicles (C) membranous urethra (D) neck of the bladder (E) ischiocavernous muscle

(E) The following structures are found in the superficial perineal space: the root of the penis and its associated muscles, the ischiocavernous and bulbospongiosus. The proximal part of the spongy urethra, superficial transverse perineal muscles, internal pudendal vessels, and branches of the pudendal nerve are also located in the superficial perineal space.

The scrotum arises from which of the following structures? (A) mesonephric ducts (B) paramesonephric ducts (C) urethral folds (D) cloacal fold (E) genital swellings

(E) The genital swellings, known in the male as the scrotal swellings, arise in the inguinal region. With further development, they move caudally, and each swelling then makes up half of the scrotum.

Which of the following muscles is the larger part and most important muscle in the pelvic floor? (A) coccygeus (B) obturator internus (C) piriformis (D) deep transverse perineal muscle (E) levator ani

(E) The levator ani, a broad muscular sheet, is the largest and most important muscle in the pelvic floor.

Which of the following structures forms the mesentery of the uterus? (A) round ligament of the uterus (B) mesosalpinx (C) pelvic diaphragm (D) endopelvic fascia (E) mesometrium

(E) The major part of the broad ligament, the mesentery of the uterus or mesometrium, is below the mesosalpinx and mesovarium.

A 62-year-old man is incapable of penile erection after rectal surgery with prostatectomy. The patient most likely has a lesion of which of the following nerves? (A) Dorsal nerve of the penis (B) Perineal nerve (C) Hypogastric nerve (D) Sacral splanchnic nerve (E) Pelvic splanchnic nerve

(E) The pelvic splanchnic nerve contains preganglionic parasympathetic fibers, whereas the sacral splanchnic nerve contains preganglionic sympathetic fibers. Parasympathetic fibers are responsible for erection, whereas sympathetic fibers are involved with ejaculation. The right and left hypogastric nerves contain primarily sympathetic fibers and visceral sensory fibers. The dorsal nerve of the penis and the perineal nerve provide sensory nerve fibers.

All of the following structures surround the corpora cavernosa and corpus spongiosum EXCEPT (A) loose areolar tissue (B) deep fascia (C) tunica albuginea (D) skin (E) tunica dartos

(E) The penis is composed of three cylindrical bodies of erectile cavernous tissue enclosed by a fibrous capsule, the tunica albuginea. Superficial to the capsule is the deep fascia of the penis (Buck's fascia). Superficial to the deep fascia is the loose areolar tissue, which lies just beneath the skin of the penis. The tunica dartos is located in the scrotum.

A 68-year-old woman with uterine carcinoma undergoes surgical resection. This cancer can spread directly to the labia majora in lymphatics that follow which of the following structures? (A) Pubic arcuate ligament (B) Suspensory ligament of the ovary (C) Cardinal (transverse cervical) ligament (D) Suspensory ligament of the clitoris (E) Round ligament of the uterus

(E) The round ligament of the uterus runs laterally from the uterus through the deep inguinal ring, inguinal canal, and superficial inguinal ring and becomes lost in the subcutaneous tissues of the labium majus. Thus, carcinoma of the uterus can spread directly to the labium majus by traveling in lymphatics that follow the ligament.

A 20-year-old woman is involved in a vehicular accident and struck on the driver's side of the automobile she is driving. She is taken to the emergency department, where physical examination shows low blood pressure and tenderness on the left midaxillary line. Upon further examination of the patient, the physician also notes a large swelling that protrudes downward and medially below the left costal margin. Which of the following abdominal organs in this patient was most likely injured? (A) Descending colon (B) Left kidney (C) Liver (D) Pancreas (E) Spleen (F) Stomach

(E) The spleen is located in the upper left quadrant of the abdomen, deep to the left costal margin. The descending colon is posterior to the midaxillary line in the retroperitoneal position, as is the left kidney. The liver is located in the upper right quadrant. The pancreas is in the retroperitoneal position in the midline. The stomach is in the upper left quadrant, but it would not create a swelling as would the damaged spleen.

A 46-year-old woman has a history of infection in her perineal region. A comprehensive examination reveals a tear of the superior boundary of the superficial perineal space. Which of the following structures would most likely be injured? (A) Pelvic diaphragm (B) Colles's fascia (C) Superficial perineal fascia (D) Deep perineal fascia (E) Perineal membrane

(E) The superior (deep) boundary of the superficial perineal space is the perineal membrane (inferior fascia of the urogenital diaphragm). Colles's fascia is the deep membranous layer of the superficial perineal fascia. The deep perineal fascia essentially divides the superficial perineal space into a superficial and deep compartment. The pelvic diaphragm consists of the levator ani and coccygeus muscles.

All of the following statements concerning the pelvic cavity are correct EXCEPT (A) It is bounded posteriorly by the coccyx. (B) It is bounded anteriorly by the pubic symphysis. (C) The pelvic inlet forms the superior boundary. (D) The pelvic outlet forms the inferior boundary. (E) The musculofascial pelvic diaphragm closes the pelvic inlet.

(E) The superior boundary of the pelvic cavity is the pelvic inlet. The pelvis is limited inferiorly by the pelvic outlet, which is closed by the musculofascial pelvic diaphragm and bounded posteriorly by the coccyx and anteriorly by the pubic symphysis.

A 65-year-old man is admitted to hospital with symptoms of an upper bowel obstruction. A CT scan reveals that a large vessel is compressing the third (transverse) portion of the duodenum. Which of the following vessels is most likely involved in the obstruction? (A) Gastroduodenal artery (B) Inferior mesenteric artery (C) Portal vein (D) Splenic artery (E) Superior mesenteric artery

(E) The superior mesenteric artery (and vein) course over the third portion of the duodenum.

The term vulva is synonymous with which of the following structures? (A) mons pubis (B) labia majora (C) vestibule (D) clitoris (E) pudendum

(E) The synonymous terms vulva and pudendum include the clitoris, vestibule of the vagina, bulbs of vestibule, and greater vestibular gland.

The space between the labia minora, the vestibule, contains all of the following structures EXCEPT (A) urethral opening (B) opening of the vagina (C) ducts of the greater vestibular glands (D) ducts of the lesser vestibular glands (E) bulbs of the vestibule

(E) The vestibule is the space between the labia minora containing the openings of the urethra, vagina, and ducts of the greater and lesser vestibular glands. The greater vestibular glands are round or oval and are partly overlapped posteriorly by bulbs of the vestibule and, like the bulbs, are partially surrounded by the bulbospongiosus muscles.

Which of the following structures separates the bladder from the pubic bones in females? (A) rectouterine fold (B) vesicouterine pouch (C) trigone of the bladder (D) median umbilical fold (E) retropubic space

(E) When empty, the adult male or female urinary bladder is in the lesser pelvis, lying posterior and slightly superior to the pubic bones. It is separated from these bones by the potential retropubic space and lies inferior to the peritoneum, where it rests on the pelvic floor.


Set pelajaran terkait

AP Psych 12 - Psychological Disorders

View Set

Chapter 62: Management of Patients With Burn Injury

View Set

Chapter 29- Relationship of Principle and Agent

View Set

Combo USMLE/ Shelf / PANCE/ EOR General Surgery Mega Review

View Set